Intégrabilité

ThéorèmeIntégrabilité et convergence uniforme

Soit (\(f_{n}\)) une suite de fonctions continues sur \([a, b]\) qui converge uniformément vers une fonction \(f\) (qui est donc continue sur \([a, b]\) par application du théorème 5). On note \(F_{n}\) la primitive de \(f_{n}\) sur \([a, b]\) nulle en \(a\) et \(F\) la primitive de \(f\) sur \([a, b]\) nulle en \(a\).

Alors, (\(F_{n}\)) converge uniformément sur \([a, b]\) vers \(F\).

Démonstration

Pour \(\varepsilon > 0\) fixé, puisque (\(f_{n}\)) converge uniformément vers \(f\) sur \([a, b]\), on peut trouver \(N\) dans \(\mathbb{N}\) tel que, pour \(n \geq N\) et pour \(t \in [a, b]\), \(\left| f_{n}(t) - f(t)\right| \leq \frac{\varepsilon}{b - a}\).

En intégrant les deux membres de cette inégalité :

\(\overset{x}{\underset{a}{\int}}{\left| f_{n}(t) - f(t)\right| \textrm{dt}}~~\leq~~\overset{x}{\underset{a}{\int}} \frac{\varepsilon}{b - a} \textrm{dt}~~=~~\frac{x - a}{b - a} \varepsilon~~\leq~~\varepsilon\)

donc, \(\left| \overset{x}{\underset{a}{\int}}~~{\left( f_{n}(t) - f(t)\right) \textrm{dt}} \right|~~\leq~~\overset{x}{\underset{a}{\int}}~~{\left| f_{n}(t) - f(t)\right| \textrm{dt}}~~\leq~~\varepsilon\) dès que \(n \geq N\)

Ce qui montre que \(\underset{n \rightarrow +\infty}{\textrm{lim}} \left( \overset{x}{\underset{a}{\int}}~~\left( f_{n}(t) - f(t)\right)~\textrm{dt} \right) = 0\), et donc que : \(\underset{n \rightarrow +\infty}{\textrm{lim}} \left( \overset{x}{\underset{a}{\int}}~~f_{n}(t)~\textrm{dt} \right) = \overset{x}{\underset{a}{\int}}~~f(t)~\textrm{dt}\).

Donc : \(\underset{n \rightarrow +\infty}{\textrm{lim}} F_{n}(x) = F(x)\).

La suite (\(F_{n}\)) converge donc simplement vers \(F\) sur \([a, b]\).

De plus, l'entier \(N\) ne dépend pas de \(x\) (il ne dépend que de \(b\)), donc la convergence est uniforme.

ThéorèmeCorollaire ; Intégration

Sous les mêmes conditions que le théorème précédent, on a : 

\(\forall x \in [a, b] \quad \underset{n \rightarrow +\infty}{\textrm{lim}} \left( \overset{x}{\underset{a}{\int}}~~f_{n}(t)~\textrm{dt} \right) = \overset{x}{\underset{a}{\int}}~~f(t)~\textrm{dt}\)

La convergence uniforme est une condition suffisante pour intervertir \(\textrm{lim}\) et \(\int\).

ExempleExemple 10

\(f_{n}(x) = n x e^{-n~\left( \frac{x^{2}}{2}\right)}\) et \(A = [0, a]\).

Pour \(x = 0\), \(f_{n}(x) = 0\) pour tout \(n\).

Pour \(0 < x \leq a\), \(f_{n}(x) = \frac{nx}{e^{n~\left(\frac{x^{2}}{2}\right)}}\) et donc \(\underset{n \rightarrow + \infty}{\textrm{lim}} f_{n}(x) = 0\).

La suite (\(f_{n}\)) converge donc simplement vers \(\overset{\sim}{0}\) sur \([0, a]\).

Ceci montre donc que la convergence de la suite (\(f_{n}\)) vers \(\overset{\sim}{0}\) n'est pas uniforme sur \([0, a]\).

RemarqueRemarque 1

L'interversion est parfois possible sans qu'il y ait convergence uniforme ; autrement dit, la convergence uniforme n'est pas une condition nécessaire pour pouvoir intervertir \(\textrm{lim}\) et \(\int\).

L'exemple 3 de l'introduction nous le montre : \(\overset{1}{\underset{0}{\int}}~~t^{n}~\textrm{dt} = \frac{1}{n + 1} \underset{n \rightarrow +\infty}{\longrightarrow} 0\) ; \((f_{n}) \overset{\textrm{CVS}}{\underset{[0, 1]}{\longrightarrow}} \overset{\sim}{0}\), et \(\overset{1}{\underset{0}{\int}}~~\overset{\sim}{0}~(t)~\textrm{dt} = 0\) alors que la convergence n'est pas uniforme comme nous l'avons vu plus haut.

RemarqueRemarque 2

Le théorème 6 (Intégrabilité et convergence uniforme) est faux sur un intervalle non borné comme le montre l'exemple 11 suivant.

ExempleExemple 11

Soit (\(f_{n}\)) la suite de fonctions définies sur \([1, \infty[\) par \(f_{n}(x) = \frac{1}{nx}\). Il est facile de voir que, pour \(x \geq 1\), la suite \(\left( f_{n}(x) \right)\) tend vers 0 (donc \((f_{n}) \overset{\textrm{CVS}}{\underset{[1, +\infty[}{\longrightarrow}} \overset{\sim}{0})\).

La convergence est uniforme car \left| f_{n}(x) - \overset{\sim}{0} (x)\right| = \frac{1}{nx} \leq \frac{1}{n} sur [1, +\infty[, donc \(\underset{[1, +\infty[}{\textrm{sup}} \left(~\left| f_{n}(x) - \overset{\sim}{0}(x) \right|~\right) \leq \frac{1}{n} \underset{n \rightarrow + \infty}{\longrightarrow} 0\).

\(\overset{+ \infty}{\underset{1}{\int}}~~\overset{\sim}{0}~(t)~\textrm{dt} = 0\).

\(\overset{+ \infty}{\underset{1}{\int}}~~f_{n}(t)~\textrm{dt} = \frac{1}{n} \Big[ \textrm{ln}(t) \Big]^{+\infty}_{1} = +\infty\) donc, \(\underset{n \rightarrow +\infty}{\textrm{lim}} \left( \overset{+\infty}{\underset{1}{\int}} f_{n}(t)~\textrm{dt} \right) = + \infty\).

RemarqueRemarque 3

Dans le théorème 6, on peut affaiblir la condition "les \(f_{n}\) sont toutes continues sur \([a, b]\)" par "les \(f_{n}\) sont toutes continues par morceaux sur \([a, b]\)" ou par "les \(f_{n}\) sont toutes intégrables sur \([a, b]\)".